0
$\begingroup$

Let $p$ and $q$ be integers.

Let $f(n)$ be A007814, the exponent of the highest power of $2$ dividing $n$, a.k.a. the binary carry sequence, the ruler sequence, or the $2$-adic valuation of $n$.

Then we have an integer sequence given by \begin{align} a(0)=a(1)&=1\\ a(2n)& = pa(n)+qa(2n-2^{f(n)})\\ a(2n+1) &= a(n-2^{f(n)}) \end{align}

Let $$s(n) = \sum\limits_{k=2^{n-1}}^{2^{n}-1}a(k), s(0)=1$$ then I conjecture that $$\sum\limits_{k=0}^{\infty}s(k)x^k=\frac{1}{1-x}+\sum\limits_{k=2}^{\infty}\left(\prod\limits_{j=2}^{k}(q^{j-1}+p\sum\limits_{r=0}^{j-2}q^r)\right)\frac{x^{2(k-1)}\left(1-(-1+q^{k-1}+p\sum\limits_{s=0}^{k-2}q^s)x\right)}{(1-x)\prod\limits_{i=2}^{k}\left(1-(q^{i-1}+p\sum\limits_{t=0}^{i-2}q^t)x\right)^2}$$ There also exist finite variant: $$\sum\limits_{k=0}^{n}s(k)x^k=\frac{1}{1-x}+\sum\limits_{k=2}^{\left\lfloor\frac{n}{2}\right\rfloor+1}\left(\prod\limits_{j=2}^{k}(q^{j-1}+p\sum\limits_{r=0}^{j-2}q^r)\right)\frac{x^{2(k-1)}\left(1-(-1+q^{k-1}+p\sum\limits_{s=0}^{k-2}q^s)x\right)}{(1-x)\prod\limits_{i=2}^{k}\left(1-(q^{i-1}+p\sum\limits_{t=0}^{i-2}q^t)x\right)^2}$$

Is there a way to prove it?

$\endgroup$

1 Answer 1

0
$\begingroup$

As proved in this answer, if we represent $n$ as $n=2^{t_1}(1+2^{t_2+1}(1+\dots(1+2^{t_m+1}))\dots)$ with $t_j\geq 0$, then \begin{split} a(n) = P(\ell)^{t_1}\prod_{j=1}^{\ell-1} P(\ell-j)^{t_{2j}+t_{2j+1}+1}, \end{split} where $\ell:=\left\lfloor\frac{m+1}2\right\rfloor$ and $$P(k) := q^k+p\frac{q^k-1}{q-1}.$$ Notice that this does not depend on $t_m$ when $m$ is even.

Grouping the summands in $s(n)$ by the number of unit bits (very much like in this other answer), for $n\geq 1$ we have \begin{split} s(n) &= \sum_{m=1}^n\ \sum_{t_1+t_2+\dots+t_{m}=n-m}\ P(\lfloor(m+1)/2\rfloor)^{t_1} \prod_{j=1}^{\lfloor(m-1)/2\rfloor} P(\lfloor(m+1)/2\rfloor-j)^{t_{2j}+t_{2j+1}+1}\\ &= \sum_{m=1}^n\ [x^{n-m}]\ \frac1{1-P(\lfloor(m+1)/2\rfloor)x}\cdot\prod_{j=1}^{\lfloor(m-1)/2\rfloor} \frac{P(j)}{(1-P(j)x)^2}\cdot\frac{1+\frac{(-1)^m-1}2x}{1-x}\\ &= [x^n]\ \sum_{m=1}^\infty\ \frac{x^m}{1-P(\lfloor(m+1)/2\rfloor)x}\cdot\prod_{j=1}^{\lfloor(m-1)/2\rfloor} \frac{P(j)}{(1-P(j)x)^2}\cdot\frac{1+\frac{(-1)^m-1}2x}{1-x}. \end{split} That is, the generating function for $s(n)$ is $$\sum_{n\geq 0} s(n)x^n = 1+\frac1{1-x}\sum_{m=1}^\infty\ \frac{x^m(1+\frac{(-1)^m-1}2x)}{1-P(\lfloor(m+1)/2\rfloor)x}\cdot\prod_{j=1}^{\lfloor(m-1)/2\rfloor} \frac{P(j)}{(1-P(j)x)^2}.$$

Combining terms for $m=2k-1$ and $m=2k$, we can rewrite it as $$\sum_{n\geq 0} s(n)x^n = 1+\frac{1}{1-x}\sum_{k=1}^\infty\ \frac{x^{2k-1}}{1-P(k)x}\cdot\prod_{j=1}^{k-1} \frac{P(j)}{(1-P(j)x)^2},$$ which is quite close to the conjectured formula.

$\endgroup$
0

Your Answer

By clicking “Post Your Answer”, you agree to our terms of service and acknowledge that you have read and understand our privacy policy and code of conduct.

Not the answer you're looking for? Browse other questions tagged or ask your own question.